LSAT and Law School Admissions Forum

Get expert LSAT preparation and law school admissions advice from PowerScore Test Preparation.

 Administrator
PowerScore Staff
  • PowerScore Staff
  • Posts: 8916
  • Joined: Feb 02, 2011
|
#25920
Complete Question Explanation
(The complete setup for this game can be found here: lsat/viewtopic.php?t=5457)

The correct answer choice is (E)

The question stem establishes that Z lectures fourth. Since either Z or W must lecture seventh, here we can conclude that W lectures seventh:
oct12_game_2_#12_diagram_1.png
We know that V lectures before Z (third rule), and also that T and S occupy two of the first three positions on the diagram. With the first three positions fully occupied, our local setup would look like this:
oct12_game_2_#12_diagram_2.png
The setup has multiple points of uncertainty, but is nevertheless sufficient to eliminate four of the answer choices from consideration: Answer choices (A) and (B) are incorrect, because P lectures either fifth or sixth. Answer choice (C) is incorrect, because W lectures seventh. Because Y lectures either fifth or sixth, answer choice (D) cannot be true, whereas answer choice (E) could be true and is therefore correct.
You do not have the required permissions to view the files attached to this post.

Get the most out of your LSAT Prep Plus subscription.

Analyze and track your performance with our Testing and Analytics Package.